![]() |
|
|
FlashChat | Actuarial Discussion | Preliminary Exams | CAS/SOA Exams | Cyberchat | Around the World | Suggestions |
Long-Term Actuarial Math Old Exam MLC Forum |
![]() |
|
Thread Tools | Search this Thread | Display Modes |
#1
|
|||
|
|||
![]() Solution for Fall 2018 WA is here:
http://deeperunderstandingfastercalc...018_WA_sol.pdf Thanks for reading. |
#3
|
|||
|
|||
![]() For WA 4, YES; you can use the recursive reserve formula to derive the reserve at any time.
Recursive tV formula should always work regardless of whether the premium is calculated under the equivalence principle and regardless of whether the reserve is gross, net, FPT, or based any other modified reserve method. |
#4
|
||||
|
||||
![]() Quote:
__________________
APC ![]() |
#5
|
||||
|
||||
![]() Roughly graded my paper. Not all solutions are in my memory. Think im at a score of 66. Not sure if that will make the cut.
__________________
APC ![]() |
#6
|
|||
|
|||
![]() In case you are wondering why in WA 1 SOA tellls you that premiums in the first two years are lower.
This information is to support their profit signatures 84.74 for Year 1 and 80.35 Year 2. If the premiums are level for all years, then the profit signatures for Year 1 and Year 2 will not be 84.74 and 80.35 respectively. |
#8
|
||||
|
||||
![]() This thread is as about Guo's WA solutions. If you are asking about MC, the SOA's solutions include a 10p60^(01) and do not include a 11p60^(01). There is a thread about Guo's MC solutions but I'm not going to look for it. I think there are also TIA solutions, maybe not in a separate thread.
|
#9
|
|||
|
|||
![]() My solution to Fall 2018 LTAM MC Q9 has a typo in the recursive formula.
In the recursive formula, the correct term should be 10p60^(01). However, I wrote 11p60^(01). I must be drunk that day! However, my numerical calculation is still correct. Thanks BartimaeusOfUruk and Gandalf. |
![]() |
Thread Tools | Search this Thread |
Display Modes | |
|
|